Questions tagged [finite-fields]

A finite field is a field with a finite number of elements. For each prime power $q^k$, there is a unique (up to isomorphism) finite field with $q^k$ elements. Up to isomorphism, these are the only finite fields.

Filter by
Sorted by
Tagged with
2 votes
1 answer
187 views

A Vandermonde-type system

For a prime $p$ and $a_1,\dotsc,a_n\in\mathbb F_p^\times$, consider the system of equations $$ \begin{cases} \begin{align} a_1 + \dotsb + a_n &= 0 \\ a_1x_1 + \dotsb + a_nx_n &...
Seva's user avatar
  • 22.8k
1 vote
0 answers
112 views

Nature of polynomials of the form $x^n-a$ over finite fields

I state the following theorem from Serge Lang's Book- Algebra(3rd edition). Theorem: Let $k$ be a field and $n$ an integer $\geq$ 2. Let $a\in k, a\neq 0$. Assume that for all primes $p$ such that $...
Riju's user avatar
  • 430
3 votes
2 answers
244 views

Upper bounds on the order of a number in prime fields

Let $k$ be a fixed integer and for any prime number $p$ larger than $k$, let $\text{Order}(k,p)$ be the order of $k$ in $\mathbb{F}_p$ (i.e., $\text{Order}(k,p)$ is the least integer $n$ such that $k^...
C.P.'s user avatar
  • 343
1 vote
0 answers
172 views

Imagining linear maps between finite fields

I can't imagine a right picture of a linear transformation $\mathbb{F}_{p} \mapsto \mathbb{F}_p$ or $\mathbb{F}_{p^2} \rightarrow \mathbb{F}_{p^2}$ etc (over the field $\mathbb{F}_p$) although they ...
katana_0's user avatar
  • 353
3 votes
2 answers
523 views

1/2 Wilson's theorem

During my research I came across this question : Question: What's the value of $x_p=(\dfrac{p-1}{2})! \mod p$ when $p>3$ is prime ? Remark: It's easy to see $x_p^2 \mod p=(-1)^{\dfrac{p+1}{2}} \...
Dattier's user avatar
  • 3,791
2 votes
0 answers
206 views

Satake correspondence for groups over finite field

I asked the same question in MSE, but I didn't get any answer. So I decided to post it here, too. In Langlands' program, Satake correspondence gives a correspondence between unramified ...
Seewoo Lee's user avatar
  • 1,911
7 votes
1 answer
373 views

Smoothness of the moduli space of Drinfeld modules

I'm studying the proof of Thm 1.5.1. in Laumon's "Cohomology of Drinfeld Modular Varieties". Notation: $\mathfrak{m}$ is a square zero ideal of $\mathcal{O}$ and $k=\mathcal{O}/\mathfrak{m}$. Laumon ...
bad mathematician's user avatar
12 votes
1 answer
267 views

Most points on a degree $p$ hypersurface?

Let $p$ be a prime. Let $f \in \mathbb{F}_p[x_1, \ldots, x_n]$ be a homogenous polynomial of degree $p$. Can $f$ have more than $(1-p^{-1}+p^{-2}) p^n$ zeroes in $\mathbb{F}_p^n$? Basic observations: ...
David E Speyer's user avatar
8 votes
1 answer
482 views

What is the minimum $k$ such that $A^k \equiv I$ mod p for invertible matrices?

Let $F$ be a finite field of order $p$, where $p$ is prime. For any $n\times n$ matrix $A$ that is invertible over $F$, then there would appear to exist integers $k$ such that $A^{k} = I$. My question ...
Joe Fitzsimons's user avatar
1 vote
0 answers
69 views

On equality of two quotients of a congruence subgroup

Related question: Non-torsion part of the abelianisation of congruence subgroups Let $A = \mathbb{F}_q[T]$ be the ring of polynomials with coefficients in a finite field, with $N$ a nonconstant ...
Liam Baker's user avatar
6 votes
0 answers
371 views

Is there an easy way to compute the maximum isotropic subspace over finite fields?

Given a quadratic form (or a symmetric $n \times n$ matrix $A$), an isotropic subspace is a subspace $U$ such that $$U^t A U=0,$$ If I am not mistaken, when the matrix is over reals, the maximum ...
Hao's user avatar
  • 571
5 votes
2 answers
625 views

The slope of $nx\ \%\ m$ [closed]

(This is a follow-up question to another question I asked at MSE. I edited due to an important hint from Will Sawin - see his comments below.) There will be this question at the end of this post: ...
Hans-Peter Stricker's user avatar
0 votes
0 answers
145 views

Groups implementable by finite field

I'm interested in finding all groups for which the group operation (and inverse map) may be implemented using finite field arithmetic. I've done some searching and have come across "algebraic groups",...
user135066's user avatar
3 votes
1 answer
276 views

A summation of powers defined by an equation over finite fields

Let $p$ be an odd prime and let $k\mid q$ for some positive integer numbers $k$ and $q$. Suppose that $r \in \mathbb{F}_{p^q}$ has multiplicative order $p^k-1$. For each $1\leq u \leq p^k-3$, the ...
user0410's user avatar
  • 211
9 votes
0 answers
415 views

Commuting matrix variety $[A,B]=0$ - can one geometrically explain divisibility of $F_ q$ point count by high powers of $q$?

$\DeclareMathOperator\Comm{Comm}\DeclareMathOperator\Id{Id}$Consider the variety $\Comm$ of commuting matrices $[A,B]=0$ over some field $K$. It is much studied, and interesting for various reasons. ...
Alexander Chervov's user avatar
2 votes
0 answers
172 views

On an exercise in The Probabilistic Method : random dilate of a set in a finite field

This is related to Problem $4.6$ in ``The Probabilistic Method'' by Alon and Spencer, where one essentially has to prove the following: Let $p$ be a prime, and $A$ be any subset of $\mathbb{F}_p$. ...
Aditya's user avatar
  • 21
2 votes
1 answer
275 views

Is there an analogue of the Balazard-Saias-Yor criterion for the Riemann Hypothesis for finite fields?

The Balazard-Saias-Yor criterion for the Riemann Hypothesis states that the latter is equivalent to the statement that $$\int_{\Re(s)=1/2} \frac{\log|\zeta(s)|}{|s^2|}|ds|=0$$ where $\zeta$ denotes ...
OneTwoOne's user avatar
  • 105
1 vote
0 answers
117 views

A series of conjectures on $\sum_{x=0}^{(p-1)/2}(\frac{x^5+cx^3+dx}p)$ (II)

As in Question 319254, for an odd prime $p$ and integers $c,d$ we let $$S_p(c,d):=\sum_{x=0}^{(p-1)/2}\left(\frac{x^5+cx^3+dx}p\right).$$ If $p\equiv1\pmod4$, then we obviously have \begin{align}&\...
Zhi-Wei Sun's user avatar
  • 14.5k
1 vote
1 answer
2k views

Write the algebra closure of $F_p$ as union of finite fields [closed]

This question follows Field theory by Steven Roman, Chapter 9, Exercise 20. Denote the algebraic closure of the finite field $F_q$ by $\Gamma(q)$, and let $a_n$ be any strictly increasing infinite ...
Wembley Inter's user avatar
0 votes
2 answers
281 views

Permutations of squares and finite fields

Let $S_n$ be the symmetric group of all permutations of $\{1,\ldots,n\}$, and let $$S(n)=\bigg\{\sum_{k=1}^nk^2\pi(k)^2:\ \pi\in S_n\}.$$ Motivated by Question 316142 of mine, here I ask the following ...
Zhi-Wei Sun's user avatar
  • 14.5k
6 votes
1 answer
288 views

Covering the finite plane with lines

This is, essentially, a geometrically rendered version of the question I asked a week ago, with the emphases slightly shifted; it seems more natural and appealing (to me, at least) in this form. Let ...
Seva's user avatar
  • 22.8k
1 vote
0 answers
154 views

On the set $\{\sum_{k=1}^n \lambda_ka_k:\ a_1,\ldots,a_k\ \text{are distinct elements of}\ A\}$

For a field $F$ let $p(F)=p$ if the characteristic of $F$ is a prime $p$, and $p(F)=+\infty$ if $F$ is of characteristic zero. In 2007 I considered the linear extension of the Erdos-Heilbronn ...
Zhi-Wei Sun's user avatar
  • 14.5k
4 votes
2 answers
522 views

Non-torsion part of the abelianisation of congruence subgroups

I've posted this question on math.stackexchange, but haven't gotten any responses so I'm trying here instead. Let $A = F_q[T]$ be the ring of polynomials in one variable with coefficients in a finite ...
Liam Baker's user avatar
4 votes
1 answer
319 views

Submersion implies many rational points in image?

Let $A \colon V \to W$ be a surjective linear map (defined over $\mathbb{Z}$), inducing a projection $\alpha \colon \mathbb{P}(V) \to \mathbb{P}(W)$. Let $X \subseteq \mathbb{P}(V)$ and $Y \...
darko's user avatar
  • 165
2 votes
0 answers
138 views

The growth of class number in $\mathbb{Z}_p$-extensions of function fields

Let $X$ be a curve (proper, smooth, ...) over a finite field $\mathbb F_q$ where $q$. Suppose also that $\mathbb F_q$ contains the $p$-th roots of unity, in this case we have the following (unique) ...
Asvin's user avatar
  • 7,646
2 votes
0 answers
95 views

Polynomials passing through points with tangential conditions

In corollary here http://math.mit.edu/~lguth/Exposition/erdossurvey.pdf on polynomial methods it is said "(Parameter counting) If $S\subset\mathbb F^n$ is a finite set, then there is a non-zero ...
Turbo's user avatar
  • 13.7k
6 votes
0 answers
152 views

$\mathbb{F}_q$-rational elements in unipotent classes of a finite group of Lie-type

I've tried posting this question on MSE, but didn't manage to get an answer there, so I'm trying again here. Sorry in advance if this question is trivial or trivially false. I haven't managed to find ...
kneidell's user avatar
  • 993
2 votes
1 answer
349 views

The minimum rank of a matrix over GF(2) when part of non-zero off-diagonal elements are set to be zero

Given an $n\times n$ matrix $A$, whose elements are over $GF\left(2\right)$ and all diagonal elements are $1$. There are $m\ (m\leq n^2-n)$ non-zero off-diagonal elements in $A$. If we are allowed to ...
Tang's user avatar
  • 21
6 votes
1 answer
335 views

2-Torsion in Jacobians of Curves Over Finite Fields

Let $C$ be a (smooth, projective) curve over a finite field $\mathbb{F}_q$, and let $J_C(\mathbb{F}_q)$ denote its Jacobian. Suppose the genus $g$ of $C$ is at least $1$. Question 1: Are there curves ...
Ofir Gorodetsky's user avatar
1 vote
1 answer
214 views

Factorisation of polynomials over finite field

Is there a method to factorise a polynomial, for $k \leq m$ and $a_i \in \mathbb{F}_p$, $$ 1 + t^k(1 + a_1 t + a_2 t + \ldots + a_m t^m)^k $$ as a product $$ (1 + t^k)^{x_1} \cdots (1 + t^l)^{x_l} \...
Vanya's user avatar
  • 581
13 votes
2 answers
495 views

Roots of lacunary polynomials over a finite field

If $P$ is a polynomial over the field $\mathbb F_q$ of degree at most $q-2$ with $k$ nonzero coefficients, then $P$ has at most $(1-1/k)(q-1)$ distinct nonzero roots. Does this fact have any standard ...
Seva's user avatar
  • 22.8k
6 votes
1 answer
173 views

Smallest set of nonzero vectors in $\mathbb F_2^n$ which intersects every 2-dimensional subspace

What is the smallest set of nonzero vectors in $\mathbb F_2^n$ which intersects every 2-dimensional subspace? For example, for n = 3, the set {001, 010, 011} does the job, and is minimal. For n = 4, {...
JacobS's user avatar
  • 61
1 vote
1 answer
151 views

Lifting Lang-Steinberg to DVR's in Characteristic 0

Let $A$ be a compact DVR in characteristic $0$, uniformizer $\pi$ and residue field $k$. Let $A\subset B$ be a complete DVR with the same uniformizer $\pi$ and algebraicly closed residue field $F$. ...
Estus's user avatar
  • 273
2 votes
0 answers
264 views

Intersection of two varieties in $\mathbf{F}_q^n$

Suppose we identify $\mathbf{F}_q^n$ with $\mathbf{F}_{q^n}$. Let $X_n$ be the irreducible hypersurface defined by $Nx=1$ where $N$ is the norm map. There is an analogous hypersurface $X_{n-1}$ in $\...
Sean Eberhard's user avatar
6 votes
1 answer
594 views

Upperbounding a sum of Legendre-Symbols

Let $p$ be a prime with $p\equiv 3 \mod 4$, for any $\mathcal{I} \subset \lbrace 0,...,p-1 \rbrace $ and any $\mathcal{J} \subset \lbrace 0,...,p-1 \rbrace $ with $\vert\mathcal{I}\vert \leq \sqrt{p} $...
nahila's user avatar
  • 93
1 vote
0 answers
1k views

Are the integers a vector space or algebra over "some" field or over "some" ring?

Every vector $v$ in a finite-dimensional vector space space $V$ of dimension $n$ over a field $F$ has a unique representation in terms of a basis ${\frak B} \subseteq V$, where a basis for $V$ is a ...
étale-cohomology's user avatar
4 votes
0 answers
99 views

Monoid cohomology of $\mathbb{N}$ for a linear algebraic group

Let $k$ be a finite field and $k_E:=k((X))$ denote the field of Laurent series over $k$. We define a Frobenius endomorphism on $k_E$ via $f(X)\mapsto f(X^p)$. We choose a lift $\varphi:k_E^{sep}\...
Estus's user avatar
  • 273
5 votes
2 answers
674 views

Surjectivity of norm map on subspaces of finite fields

It is basic that the norm map $N:\mathbf{F}_{q^n}^* \to \mathbf{F}_q^*$ is surjective for finite fields. In fact $N(x) = x^{(q^n-1)/(q-1)}$. How well does this simple fact extend to subspaces? A ...
Sean Eberhard's user avatar
1 vote
0 answers
375 views

Decomposition of a Matrix by Sparse Matrices

Let $\mathbb{F}$ is a field. Consider an $n \times n$ matrix $\bf A$ over $\mathbb{F}$. $\bf A$ is called sparse matrix over $\mathbb{F}$ iff the number of non-zero entities of $\bf A$ be at most ...
user0410's user avatar
  • 211
1 vote
1 answer
106 views

Notions of convergence over extensions of finite fields

Let $\displaystyle Q_p[x] = \left\{\frac{p(x)}{q(x)} \mid \, p(x),q(x) \in \mathbb{F}_p[x], \, q(x) \neq 0 \right\}$ denote the field of fractions extending $\mathbb{F}_p[x]$. If we consider the ...
Brian's user avatar
  • 1,459
3 votes
1 answer
167 views

Solving system of multivariable algebraic equations over $\mathbb Q$ by reducing over $\mathbb F_p$

I try to solve the finite system of multivariable algebraic equations with coefficients from $\mathbb Q$. It would be sufficient for me to prove that there is only finite number of solutions over $\...
Maxim's user avatar
  • 414
1 vote
1 answer
169 views

Pair of vectors multiplied by a random matrix and its inverse transpose are distributed randomly up to their dot product

Given arbitrary nonzero vectors $\vec{x}_1, \vec{y}_1, \vec{x}_2, \vec{y}_2 \in \mathbb{Z}^{n}_p$ ($p$ prime) with $\langle x_1, y_1 \rangle = \langle x_2, y_2 \rangle$, I am trying to show that: $(...
lkowalcz's user avatar
1 vote
0 answers
106 views

Reference request: Number of elliptic and hyperbolic quadratic forms of a given rank over a finite field

My question is over the finite field $\mathbf{F}_q$ of $q$ elements. It is well known that a symmetric matrix of odd rank corresponds to a parabolic quadratic form but even rank symmetric matrices ...
Singh's user avatar
  • 179
3 votes
2 answers
354 views

Alternate descriptions of finite fields

The finite field of order $p^n$ is isomorphic to $(\mathbb Z/p \mathbb Z)[X]/(P)$, where $P$ is an irreducible polynomial in $(\mathbb Z/p \mathbb Z)[X]$ of degree $n$. This describes every finite ...
Christopher King's user avatar
2 votes
0 answers
113 views

Lacunary fully reducible polynomial over a finite field

The following problem is motivated by this MO question on rich directions determined by a set of a finite plane. Problem Does there exist a constant $C$ such that for all odd primes $p$ there is a ...
Luca Ghidelli's user avatar
4 votes
1 answer
204 views

Balancing points with lines

$\newcommand{\F}{\mathbb F}$ Suppose that $p$ is a prime, and $k<p/2$ a positive integer. Consider a system of $k$ distinct directions in the affine plane $\F_p^2$, and the system of $k$ pencils ...
Seva's user avatar
  • 22.8k
0 votes
0 answers
145 views

On primitive roots of the form $5^k+10^m$ with $k$ and $m$ nonnegative integers

Let $p$ be any prime. It is well known that the set $$G_p=\{0<g<p:\ g\ \text{is a primitive root modulo}\ p\}$$ has cardinality $\varphi(p-1)$, where $\varphi$ is Euler's totient function. It is ...
Zhi-Wei Sun's user avatar
  • 14.5k
6 votes
1 answer
277 views

If number of points on a manifold is $q^n ( [n+1]_q )$ does it imply a geometric relation to $A^n (P^n)$?

Consider an algebraic manifold whose number of points is $q^n ([n+1]_q)$. Is there a geometric relation to $A^n (P^n)$? In particular, is there an equivalence in the Grothendieck ring of varieties ...
Alexander Chervov's user avatar
4 votes
0 answers
743 views

When spreading out a scheme, does the choice of max ideal matter?

I'm looking at Serre's paper How to use Finite Fields for Problems Concerning Infinite Fields. Specifically I'm trying to use the techniques in the proof of Theorem 1.2 to write out the details of the ...
Mike Pierce's user avatar
  • 1,149
7 votes
1 answer
498 views

Linear permutations commuting with $x\rightarrow x^{-1}$

Let $F = \operatorname{GF}(2^n)$ be a finite field. Define a permutation $\phi:F \rightarrow F$ by the formula $$ \phi(x) = x^{-1}, \ x\neq 0; \ \phi(0) =0. $$ We say that a permutations $\psi$ of $F$ ...
Mikhail Goltvanitsa's user avatar

1
5 6
7
8 9
16